distribution associated with a discontinuous function












1












$begingroup$


Let $fcolonmathbb{R}tomathbb{R}$ be such that, for every $ninmathbb{Z}$, $f$ is differentiable on $left(n,n+1right)$ and $n$ is a discontinuity of first kind of $f$. We define
$$T_f(phi)=int_{mathbb{R}}f(x)phi(x)dx,quadtext{where }phitext{ is a test function}.$$
I understand that, when $f$ is continuously differentiable, $T_f$ is well defined and the derivative of $T_f$ is just $T_{f'}$. However, these two do not seem clear in this case.



My attempt: Since $n$ and $n+1$ are discontinuities of first kind of $f$, we can define a new function, say $g_{n}$ such that $g_{n}$ is continuous on $[n,n+1]$ and equal to $f$ a.e. Thus $f$ is (Lebesgue) integrable on $[n,n+1]$. We then conclude that $f$ is locally integrable on $mathbb{R}$ and therefore $T_{f}$ is well defined.



My question: 1) Is the above reasoning (to show that $T_f$ is well defined) correct? And:



2) What would the derivative of $T_f$ look like in this case? I think it is not $T_{f'}$ as $f'$ is not defined at $n$.



Any help/hint is highly appreciated.










share|cite|improve this question









$endgroup$

















    1












    $begingroup$


    Let $fcolonmathbb{R}tomathbb{R}$ be such that, for every $ninmathbb{Z}$, $f$ is differentiable on $left(n,n+1right)$ and $n$ is a discontinuity of first kind of $f$. We define
    $$T_f(phi)=int_{mathbb{R}}f(x)phi(x)dx,quadtext{where }phitext{ is a test function}.$$
    I understand that, when $f$ is continuously differentiable, $T_f$ is well defined and the derivative of $T_f$ is just $T_{f'}$. However, these two do not seem clear in this case.



    My attempt: Since $n$ and $n+1$ are discontinuities of first kind of $f$, we can define a new function, say $g_{n}$ such that $g_{n}$ is continuous on $[n,n+1]$ and equal to $f$ a.e. Thus $f$ is (Lebesgue) integrable on $[n,n+1]$. We then conclude that $f$ is locally integrable on $mathbb{R}$ and therefore $T_{f}$ is well defined.



    My question: 1) Is the above reasoning (to show that $T_f$ is well defined) correct? And:



    2) What would the derivative of $T_f$ look like in this case? I think it is not $T_{f'}$ as $f'$ is not defined at $n$.



    Any help/hint is highly appreciated.










    share|cite|improve this question









    $endgroup$















      1












      1








      1





      $begingroup$


      Let $fcolonmathbb{R}tomathbb{R}$ be such that, for every $ninmathbb{Z}$, $f$ is differentiable on $left(n,n+1right)$ and $n$ is a discontinuity of first kind of $f$. We define
      $$T_f(phi)=int_{mathbb{R}}f(x)phi(x)dx,quadtext{where }phitext{ is a test function}.$$
      I understand that, when $f$ is continuously differentiable, $T_f$ is well defined and the derivative of $T_f$ is just $T_{f'}$. However, these two do not seem clear in this case.



      My attempt: Since $n$ and $n+1$ are discontinuities of first kind of $f$, we can define a new function, say $g_{n}$ such that $g_{n}$ is continuous on $[n,n+1]$ and equal to $f$ a.e. Thus $f$ is (Lebesgue) integrable on $[n,n+1]$. We then conclude that $f$ is locally integrable on $mathbb{R}$ and therefore $T_{f}$ is well defined.



      My question: 1) Is the above reasoning (to show that $T_f$ is well defined) correct? And:



      2) What would the derivative of $T_f$ look like in this case? I think it is not $T_{f'}$ as $f'$ is not defined at $n$.



      Any help/hint is highly appreciated.










      share|cite|improve this question









      $endgroup$




      Let $fcolonmathbb{R}tomathbb{R}$ be such that, for every $ninmathbb{Z}$, $f$ is differentiable on $left(n,n+1right)$ and $n$ is a discontinuity of first kind of $f$. We define
      $$T_f(phi)=int_{mathbb{R}}f(x)phi(x)dx,quadtext{where }phitext{ is a test function}.$$
      I understand that, when $f$ is continuously differentiable, $T_f$ is well defined and the derivative of $T_f$ is just $T_{f'}$. However, these two do not seem clear in this case.



      My attempt: Since $n$ and $n+1$ are discontinuities of first kind of $f$, we can define a new function, say $g_{n}$ such that $g_{n}$ is continuous on $[n,n+1]$ and equal to $f$ a.e. Thus $f$ is (Lebesgue) integrable on $[n,n+1]$. We then conclude that $f$ is locally integrable on $mathbb{R}$ and therefore $T_{f}$ is well defined.



      My question: 1) Is the above reasoning (to show that $T_f$ is well defined) correct? And:



      2) What would the derivative of $T_f$ look like in this case? I think it is not $T_{f'}$ as $f'$ is not defined at $n$.



      Any help/hint is highly appreciated.







      functional-analysis derivatives distribution-theory topological-vector-spaces discontinuous-functions






      share|cite|improve this question













      share|cite|improve this question











      share|cite|improve this question




      share|cite|improve this question










      asked Nov 27 '18 at 3:59









      weirdoweirdo

      420210




      420210






















          2 Answers
          2






          active

          oldest

          votes


















          1












          $begingroup$

          If $f$ is absolutely continuous on each segment $[n, n + 1]$ when defined to take the values $f(n + 0)$ and $f((n + 1) - 0)$ at the endpoints, integration by parts shows that the distributional derivative is
          $$f'(x) + sum_n (f(n + 0) - f(n - 0)) delta(x - n),$$
          where $f'(x)$ is the ordinary derivative. If the conditions for integration by parts do not hold, the distributional derivative may not be representable in this form. Also, if the test functions do not have finite support, it is necessary to add a condition on how fast $f$ can grow.






          share|cite|improve this answer











          $endgroup$





















            2












            $begingroup$

            It is important to use the fact that $f$ has left and right limits at integer points. Otherwise $f$ need not be locally integrable and $T_f$ is not a well defined distribution. The fact that these limits exist implies that it is bounded in $(n,n+1)$ and hence it is locally integrable. Any locally integrable function defines a distribution. In 2) there is no guarantee that $f'$ is even locally integrable so $T_f'$ cannot be written in terms of $f'$ without extra assumptions.






            share|cite|improve this answer











            $endgroup$













            • $begingroup$
              For the Heaviside function $f(x)=1$ for $xge 0$ and $=0$ else, the derivative is $0$ a.e. but $T_f'=delta_0$ and hence different from $T_{f'}$.
              $endgroup$
              – Jochen
              Nov 27 '18 at 7:44










            • $begingroup$
              @Jochen Thank you. I have omitted the last part of my answer.
              $endgroup$
              – Kavi Rama Murthy
              Nov 27 '18 at 7:46










            • $begingroup$
              @KaviRamaMurthy So my attempt to show that $f$ is locally integrable was correct? I was not sure which result we can use to deduce that, if $f$ is integrable on every interval $[n,n+1]$, then $f$ is integrable on each of the finite union of these sets...Perhaps I forgot something in measure theory
              $endgroup$
              – weirdo
              Nov 27 '18 at 16:41






            • 1




              $begingroup$
              @weirdo $fI_{cup E_i}=sum fI_{E_i}$ if $E_i$'s are disjoint. Finite sum of integrable functions is integrable.
              $endgroup$
              – Kavi Rama Murthy
              Nov 27 '18 at 23:08













            Your Answer





            StackExchange.ifUsing("editor", function () {
            return StackExchange.using("mathjaxEditing", function () {
            StackExchange.MarkdownEditor.creationCallbacks.add(function (editor, postfix) {
            StackExchange.mathjaxEditing.prepareWmdForMathJax(editor, postfix, [["$", "$"], ["\\(","\\)"]]);
            });
            });
            }, "mathjax-editing");

            StackExchange.ready(function() {
            var channelOptions = {
            tags: "".split(" "),
            id: "69"
            };
            initTagRenderer("".split(" "), "".split(" "), channelOptions);

            StackExchange.using("externalEditor", function() {
            // Have to fire editor after snippets, if snippets enabled
            if (StackExchange.settings.snippets.snippetsEnabled) {
            StackExchange.using("snippets", function() {
            createEditor();
            });
            }
            else {
            createEditor();
            }
            });

            function createEditor() {
            StackExchange.prepareEditor({
            heartbeatType: 'answer',
            autoActivateHeartbeat: false,
            convertImagesToLinks: true,
            noModals: true,
            showLowRepImageUploadWarning: true,
            reputationToPostImages: 10,
            bindNavPrevention: true,
            postfix: "",
            imageUploader: {
            brandingHtml: "Powered by u003ca class="icon-imgur-white" href="https://imgur.com/"u003eu003c/au003e",
            contentPolicyHtml: "User contributions licensed under u003ca href="https://creativecommons.org/licenses/by-sa/3.0/"u003ecc by-sa 3.0 with attribution requiredu003c/au003e u003ca href="https://stackoverflow.com/legal/content-policy"u003e(content policy)u003c/au003e",
            allowUrls: true
            },
            noCode: true, onDemand: true,
            discardSelector: ".discard-answer"
            ,immediatelyShowMarkdownHelp:true
            });


            }
            });














            draft saved

            draft discarded


















            StackExchange.ready(
            function () {
            StackExchange.openid.initPostLogin('.new-post-login', 'https%3a%2f%2fmath.stackexchange.com%2fquestions%2f3015318%2fdistribution-associated-with-a-discontinuous-function%23new-answer', 'question_page');
            }
            );

            Post as a guest















            Required, but never shown

























            2 Answers
            2






            active

            oldest

            votes








            2 Answers
            2






            active

            oldest

            votes









            active

            oldest

            votes






            active

            oldest

            votes









            1












            $begingroup$

            If $f$ is absolutely continuous on each segment $[n, n + 1]$ when defined to take the values $f(n + 0)$ and $f((n + 1) - 0)$ at the endpoints, integration by parts shows that the distributional derivative is
            $$f'(x) + sum_n (f(n + 0) - f(n - 0)) delta(x - n),$$
            where $f'(x)$ is the ordinary derivative. If the conditions for integration by parts do not hold, the distributional derivative may not be representable in this form. Also, if the test functions do not have finite support, it is necessary to add a condition on how fast $f$ can grow.






            share|cite|improve this answer











            $endgroup$


















              1












              $begingroup$

              If $f$ is absolutely continuous on each segment $[n, n + 1]$ when defined to take the values $f(n + 0)$ and $f((n + 1) - 0)$ at the endpoints, integration by parts shows that the distributional derivative is
              $$f'(x) + sum_n (f(n + 0) - f(n - 0)) delta(x - n),$$
              where $f'(x)$ is the ordinary derivative. If the conditions for integration by parts do not hold, the distributional derivative may not be representable in this form. Also, if the test functions do not have finite support, it is necessary to add a condition on how fast $f$ can grow.






              share|cite|improve this answer











              $endgroup$
















                1












                1








                1





                $begingroup$

                If $f$ is absolutely continuous on each segment $[n, n + 1]$ when defined to take the values $f(n + 0)$ and $f((n + 1) - 0)$ at the endpoints, integration by parts shows that the distributional derivative is
                $$f'(x) + sum_n (f(n + 0) - f(n - 0)) delta(x - n),$$
                where $f'(x)$ is the ordinary derivative. If the conditions for integration by parts do not hold, the distributional derivative may not be representable in this form. Also, if the test functions do not have finite support, it is necessary to add a condition on how fast $f$ can grow.






                share|cite|improve this answer











                $endgroup$



                If $f$ is absolutely continuous on each segment $[n, n + 1]$ when defined to take the values $f(n + 0)$ and $f((n + 1) - 0)$ at the endpoints, integration by parts shows that the distributional derivative is
                $$f'(x) + sum_n (f(n + 0) - f(n - 0)) delta(x - n),$$
                where $f'(x)$ is the ordinary derivative. If the conditions for integration by parts do not hold, the distributional derivative may not be representable in this form. Also, if the test functions do not have finite support, it is necessary to add a condition on how fast $f$ can grow.







                share|cite|improve this answer














                share|cite|improve this answer



                share|cite|improve this answer








                edited Dec 7 '18 at 15:39

























                answered Dec 1 '18 at 6:42









                MaximMaxim

                5,0881219




                5,0881219























                    2












                    $begingroup$

                    It is important to use the fact that $f$ has left and right limits at integer points. Otherwise $f$ need not be locally integrable and $T_f$ is not a well defined distribution. The fact that these limits exist implies that it is bounded in $(n,n+1)$ and hence it is locally integrable. Any locally integrable function defines a distribution. In 2) there is no guarantee that $f'$ is even locally integrable so $T_f'$ cannot be written in terms of $f'$ without extra assumptions.






                    share|cite|improve this answer











                    $endgroup$













                    • $begingroup$
                      For the Heaviside function $f(x)=1$ for $xge 0$ and $=0$ else, the derivative is $0$ a.e. but $T_f'=delta_0$ and hence different from $T_{f'}$.
                      $endgroup$
                      – Jochen
                      Nov 27 '18 at 7:44










                    • $begingroup$
                      @Jochen Thank you. I have omitted the last part of my answer.
                      $endgroup$
                      – Kavi Rama Murthy
                      Nov 27 '18 at 7:46










                    • $begingroup$
                      @KaviRamaMurthy So my attempt to show that $f$ is locally integrable was correct? I was not sure which result we can use to deduce that, if $f$ is integrable on every interval $[n,n+1]$, then $f$ is integrable on each of the finite union of these sets...Perhaps I forgot something in measure theory
                      $endgroup$
                      – weirdo
                      Nov 27 '18 at 16:41






                    • 1




                      $begingroup$
                      @weirdo $fI_{cup E_i}=sum fI_{E_i}$ if $E_i$'s are disjoint. Finite sum of integrable functions is integrable.
                      $endgroup$
                      – Kavi Rama Murthy
                      Nov 27 '18 at 23:08


















                    2












                    $begingroup$

                    It is important to use the fact that $f$ has left and right limits at integer points. Otherwise $f$ need not be locally integrable and $T_f$ is not a well defined distribution. The fact that these limits exist implies that it is bounded in $(n,n+1)$ and hence it is locally integrable. Any locally integrable function defines a distribution. In 2) there is no guarantee that $f'$ is even locally integrable so $T_f'$ cannot be written in terms of $f'$ without extra assumptions.






                    share|cite|improve this answer











                    $endgroup$













                    • $begingroup$
                      For the Heaviside function $f(x)=1$ for $xge 0$ and $=0$ else, the derivative is $0$ a.e. but $T_f'=delta_0$ and hence different from $T_{f'}$.
                      $endgroup$
                      – Jochen
                      Nov 27 '18 at 7:44










                    • $begingroup$
                      @Jochen Thank you. I have omitted the last part of my answer.
                      $endgroup$
                      – Kavi Rama Murthy
                      Nov 27 '18 at 7:46










                    • $begingroup$
                      @KaviRamaMurthy So my attempt to show that $f$ is locally integrable was correct? I was not sure which result we can use to deduce that, if $f$ is integrable on every interval $[n,n+1]$, then $f$ is integrable on each of the finite union of these sets...Perhaps I forgot something in measure theory
                      $endgroup$
                      – weirdo
                      Nov 27 '18 at 16:41






                    • 1




                      $begingroup$
                      @weirdo $fI_{cup E_i}=sum fI_{E_i}$ if $E_i$'s are disjoint. Finite sum of integrable functions is integrable.
                      $endgroup$
                      – Kavi Rama Murthy
                      Nov 27 '18 at 23:08
















                    2












                    2








                    2





                    $begingroup$

                    It is important to use the fact that $f$ has left and right limits at integer points. Otherwise $f$ need not be locally integrable and $T_f$ is not a well defined distribution. The fact that these limits exist implies that it is bounded in $(n,n+1)$ and hence it is locally integrable. Any locally integrable function defines a distribution. In 2) there is no guarantee that $f'$ is even locally integrable so $T_f'$ cannot be written in terms of $f'$ without extra assumptions.






                    share|cite|improve this answer











                    $endgroup$



                    It is important to use the fact that $f$ has left and right limits at integer points. Otherwise $f$ need not be locally integrable and $T_f$ is not a well defined distribution. The fact that these limits exist implies that it is bounded in $(n,n+1)$ and hence it is locally integrable. Any locally integrable function defines a distribution. In 2) there is no guarantee that $f'$ is even locally integrable so $T_f'$ cannot be written in terms of $f'$ without extra assumptions.







                    share|cite|improve this answer














                    share|cite|improve this answer



                    share|cite|improve this answer








                    edited Nov 27 '18 at 7:46

























                    answered Nov 27 '18 at 6:18









                    Kavi Rama MurthyKavi Rama Murthy

                    55.9k42158




                    55.9k42158












                    • $begingroup$
                      For the Heaviside function $f(x)=1$ for $xge 0$ and $=0$ else, the derivative is $0$ a.e. but $T_f'=delta_0$ and hence different from $T_{f'}$.
                      $endgroup$
                      – Jochen
                      Nov 27 '18 at 7:44










                    • $begingroup$
                      @Jochen Thank you. I have omitted the last part of my answer.
                      $endgroup$
                      – Kavi Rama Murthy
                      Nov 27 '18 at 7:46










                    • $begingroup$
                      @KaviRamaMurthy So my attempt to show that $f$ is locally integrable was correct? I was not sure which result we can use to deduce that, if $f$ is integrable on every interval $[n,n+1]$, then $f$ is integrable on each of the finite union of these sets...Perhaps I forgot something in measure theory
                      $endgroup$
                      – weirdo
                      Nov 27 '18 at 16:41






                    • 1




                      $begingroup$
                      @weirdo $fI_{cup E_i}=sum fI_{E_i}$ if $E_i$'s are disjoint. Finite sum of integrable functions is integrable.
                      $endgroup$
                      – Kavi Rama Murthy
                      Nov 27 '18 at 23:08




















                    • $begingroup$
                      For the Heaviside function $f(x)=1$ for $xge 0$ and $=0$ else, the derivative is $0$ a.e. but $T_f'=delta_0$ and hence different from $T_{f'}$.
                      $endgroup$
                      – Jochen
                      Nov 27 '18 at 7:44










                    • $begingroup$
                      @Jochen Thank you. I have omitted the last part of my answer.
                      $endgroup$
                      – Kavi Rama Murthy
                      Nov 27 '18 at 7:46










                    • $begingroup$
                      @KaviRamaMurthy So my attempt to show that $f$ is locally integrable was correct? I was not sure which result we can use to deduce that, if $f$ is integrable on every interval $[n,n+1]$, then $f$ is integrable on each of the finite union of these sets...Perhaps I forgot something in measure theory
                      $endgroup$
                      – weirdo
                      Nov 27 '18 at 16:41






                    • 1




                      $begingroup$
                      @weirdo $fI_{cup E_i}=sum fI_{E_i}$ if $E_i$'s are disjoint. Finite sum of integrable functions is integrable.
                      $endgroup$
                      – Kavi Rama Murthy
                      Nov 27 '18 at 23:08


















                    $begingroup$
                    For the Heaviside function $f(x)=1$ for $xge 0$ and $=0$ else, the derivative is $0$ a.e. but $T_f'=delta_0$ and hence different from $T_{f'}$.
                    $endgroup$
                    – Jochen
                    Nov 27 '18 at 7:44




                    $begingroup$
                    For the Heaviside function $f(x)=1$ for $xge 0$ and $=0$ else, the derivative is $0$ a.e. but $T_f'=delta_0$ and hence different from $T_{f'}$.
                    $endgroup$
                    – Jochen
                    Nov 27 '18 at 7:44












                    $begingroup$
                    @Jochen Thank you. I have omitted the last part of my answer.
                    $endgroup$
                    – Kavi Rama Murthy
                    Nov 27 '18 at 7:46




                    $begingroup$
                    @Jochen Thank you. I have omitted the last part of my answer.
                    $endgroup$
                    – Kavi Rama Murthy
                    Nov 27 '18 at 7:46












                    $begingroup$
                    @KaviRamaMurthy So my attempt to show that $f$ is locally integrable was correct? I was not sure which result we can use to deduce that, if $f$ is integrable on every interval $[n,n+1]$, then $f$ is integrable on each of the finite union of these sets...Perhaps I forgot something in measure theory
                    $endgroup$
                    – weirdo
                    Nov 27 '18 at 16:41




                    $begingroup$
                    @KaviRamaMurthy So my attempt to show that $f$ is locally integrable was correct? I was not sure which result we can use to deduce that, if $f$ is integrable on every interval $[n,n+1]$, then $f$ is integrable on each of the finite union of these sets...Perhaps I forgot something in measure theory
                    $endgroup$
                    – weirdo
                    Nov 27 '18 at 16:41




                    1




                    1




                    $begingroup$
                    @weirdo $fI_{cup E_i}=sum fI_{E_i}$ if $E_i$'s are disjoint. Finite sum of integrable functions is integrable.
                    $endgroup$
                    – Kavi Rama Murthy
                    Nov 27 '18 at 23:08






                    $begingroup$
                    @weirdo $fI_{cup E_i}=sum fI_{E_i}$ if $E_i$'s are disjoint. Finite sum of integrable functions is integrable.
                    $endgroup$
                    – Kavi Rama Murthy
                    Nov 27 '18 at 23:08




















                    draft saved

                    draft discarded




















































                    Thanks for contributing an answer to Mathematics Stack Exchange!


                    • Please be sure to answer the question. Provide details and share your research!

                    But avoid



                    • Asking for help, clarification, or responding to other answers.

                    • Making statements based on opinion; back them up with references or personal experience.


                    Use MathJax to format equations. MathJax reference.


                    To learn more, see our tips on writing great answers.




                    draft saved


                    draft discarded














                    StackExchange.ready(
                    function () {
                    StackExchange.openid.initPostLogin('.new-post-login', 'https%3a%2f%2fmath.stackexchange.com%2fquestions%2f3015318%2fdistribution-associated-with-a-discontinuous-function%23new-answer', 'question_page');
                    }
                    );

                    Post as a guest















                    Required, but never shown





















































                    Required, but never shown














                    Required, but never shown












                    Required, but never shown







                    Required, but never shown

































                    Required, but never shown














                    Required, but never shown












                    Required, but never shown







                    Required, but never shown







                    Popular posts from this blog

                    Biblatex bibliography style without URLs when DOI exists (in Overleaf with Zotero bibliography)

                    ComboBox Display Member on multiple fields

                    Is it possible to collect Nectar points via Trainline?